Finding a Basis for a Submodule of Z^3: A Linear Algebra Homework Problem

  • Thread starter Thread starter pivoxa15
  • Start date Start date
  • Tags Tags
    Basis
Click For Summary
SUMMARY

The discussion focuses on finding a basis for the submodule of the Z-module Z^3 generated by the vectors {(2,3,1), (3,4,0), (3,4,6), (5,1,4)}. The proposed solution involves constructing a matrix with these vectors as columns and performing row reduction to identify linear dependencies. By expressing one vector as a linear combination of the others, it is determined that three vectors form a basis for the submodule, as they are linearly independent and span the space.

PREREQUISITES
  • Understanding of Z-modules and their properties
  • Familiarity with linear combinations and linear independence
  • Proficiency in matrix row reduction techniques
  • Knowledge of integer coefficients in linear algebra
NEXT STEPS
  • Study the properties of Z-modules in abstract algebra
  • Learn about row reduction methods for matrices
  • Explore linear independence and spanning sets in vector spaces
  • Investigate integer linear combinations and their applications
USEFUL FOR

Students of linear algebra, mathematicians working with modules, and anyone interested in the applications of linear combinations in Z-modules.

pivoxa15
Messages
2,250
Reaction score
1

Homework Statement


How can I find a basis for a submodule of (the Z-module) Z^3 that is generated by the elements {(2,3,1), (3,4,0), (3,4,6) and (5,1,4)}"

The Attempt at a Solution


Would one way be putting each vector as columns in a matrix and row reduce. Except I got a set from the columns which could not even generate the vectors in the set above.
 
Physics news on Phys.org
As happens from time to time, I get an idea for a problem while or just after I finish typing it. And rarer does the idea actually turn out to be correct. This time I may have found the solution.

Make a linear combination of these 4 vectors equal 0 and find the coefficients for one of them in terms of the other 4 by making them into row reduced echelon form although always leaving the entries as integers. Hence one vector is made redundant. The remaining 3 form a basis as it is not linearly independent and will span the submodule.
 
Question: A clock's minute hand has length 4 and its hour hand has length 3. What is the distance between the tips at the moment when it is increasing most rapidly?(Putnam Exam Question) Answer: Making assumption that both the hands moves at constant angular velocities, the answer is ## \sqrt{7} .## But don't you think this assumption is somewhat doubtful and wrong?

Similar threads

Replies
4
Views
2K
  • · Replies 6 ·
Replies
6
Views
1K
  • · Replies 3 ·
Replies
3
Views
3K
  • · Replies 3 ·
Replies
3
Views
2K
  • · Replies 3 ·
Replies
3
Views
2K
Replies
5
Views
2K
  • · Replies 15 ·
Replies
15
Views
3K
  • · Replies 1 ·
Replies
1
Views
2K
Replies
2
Views
3K
Replies
2
Views
2K